17
$\begingroup$

Take an infinite sequence of nonnegative integers $(a_n)_{n\geq0}$, and let $s_n=\sum_{k=0}^na_n$. Define $$ \begin{split} S &=\left(\tfrac49\right)^{a_0}\left(1+\tfrac13\left(\tfrac49\right)^{a_1}\left(1+\tfrac13\left(\tfrac49\right)^{a_2}(1+\cdots)\right)\right)\\&=\sum_{n=0}^\infty\big(\tfrac13\big)^n\big(\tfrac49\big)^{s_n}.\end{split} $$ Is $S$ rational only if the sequence $(a_n)$ is eventually periodic? The converse clearly holds, as any eventually-periodic $(a_n)$ leads to a rational $S$, but there's no trivial way to show that every aperiodic sequence leads to an irrational $S$.

Perhaps some argument could be used to show that each rational number cannot be the limit of the partial sums, except in the periodic case. But I'm not sure how this could be done generically, and some googling hasn't led me to similar arguments in the literature.

This question arises from a dynamical system I'm looking at, supported on a fractal subset of $[0,1]$ formed by the IFS $\big(x\to\tfrac49x,x\to\tfrac13x+\tfrac23\big)$. I want to characterize all rational points that lie in this set, but I would have to show that every such point corresponds to an eventually-periodic orbit, i.e., $(a_n)$ sequence.

$\endgroup$
10
  • 1
    $\begingroup$ This comment might be completely irrelevant, but I think this paper forces stuff to be rational by hand and maybe you can do the same. $\endgroup$ Commented Jun 12 at 9:23
  • $\begingroup$ @mathworker21 That paper seems more focused on the construction of particular sequences that create rational sums; it doesn't seem to have so much on nonexistence. In this case, the terms decay too slowly to make the typical "too-good approximation" arguments. (In fact, replacing $\frac49$ with $\frac23$ gives every $S\in[0,\frac32]$ a valid representation: the problem would be showing that aperiodic representations of rationals hit the wrong parity at some point.) $\endgroup$ Commented Jun 12 at 19:43
  • 1
    $\begingroup$ Just to give a feel for how subtle the question is: if you replace 1/3 and 4/9 with two rational numbers whose product is strictly between 1/2 and 1, then the answer is no: there are aperiodic sequences that produce rational numbers (this can be deduced for example using prop 2.1 of irif.fr/~steiner/AFSS.pdf). 1/3 and 4/9 are a bit too small for this approach to carry over directly, so the claim may still be true in this case, but it's in some sense very close to being false. $\endgroup$ Commented Jun 18 at 0:05
  • $\begingroup$ (which is where mathworker21's comment comes in - it's entirely plausible that you might be able to cook up a counterexample) $\endgroup$ Commented Jun 18 at 0:21
  • $\begingroup$ @JonathanLove Hmm, my impression is that the 'closeness' is somewhat of an illusion, since the situations are in entirely different regimes. If we look at the IFS this comes from, it has zero measure just like the ordinary Cantor set, so we intuitively shouldn't expect it to contain any points it doesn't strictly have to. In particular, if we take any two contraction factors $x,y$ with $0<x+y<1$, then we have $xy<\frac14$: your statement wouldn't apply to anything with zero measure. $\endgroup$ Commented Jun 18 at 1:02

1 Answer 1

3
$\begingroup$

This doesn't come close to resolving the problem, but here's a small subproblem that might be useful to consider. For convenience, when I say "aperiodic" I mean "not eventually periodic."

Pick an arbitrary subset $T\subseteq\mathbb{N}$ and define $$s_n=\left\{\begin{array}{ll} n+1 & \text{ if }n\in T,\\ n & \text{ if }n\notin T. \end{array}\right.$$ This sequence is non-decreasing and so we can recover a valid sequence of non-negative integers $a_n=s_n-s_{n-1}$. For general $T$ (i.e. $T$ that is not the union of a finite set and finitely many arithmetic sequences), $a_n$ is aperiodic. Now \begin{align*} S&=\sum_{n=0}^\infty(\tfrac 13)^n(\tfrac 49)^{s_n}\\ &=\sum_{n=0}^\infty(\tfrac 13)^n(\tfrac 49)^{n}+\sum_{n\in T}(\tfrac 13)^n\left((\tfrac 49)^{n+1}-(\tfrac 49)^{n}\right)\\ &=\frac{27}{23}-\frac59\sum_{n\in T}(\tfrac {4}{27})^n, \end{align*} so $S$ is rational if and only if $\sum_{n\in T}(\tfrac {4}{27})^n$ is rational. In other words, if the original question has an affirmative answer, then the following must hold: For any base-$\frac{27}{4}$ expansion using only $0$ and $1$, the value is rational if and only if the expansion is eventually periodic.

More generally, while the original problem has two scaling factors, we can restrict to a problem with a single scaling factor by starting with a periodic sequence $a_n$ (which will allow us to combine the two scaling factors into a single product factor) and then introducing small aperiodic perturbations. Only a very narrow class of aperiodic sequences arise in this way, but if the claim is true in general, it certainly must be true for this narrow class.


Now to illustrate some of the difficulties that arise even for the restricted problem. Schmidt 1980 Theorem 2.4 proves that if the greedy $\beta$-expansion of every rational number in $[0,1)$ is eventually periodic, then $\beta$ is an algebraic integer. (In an earlier comment I cited a different paper, but that was a mistake: the result I cited there is only about purely periodic expansions.) In other words, for any non-integer rational number $\beta>1$, there exists a rational number that has an aperiodic base-$\beta$ expansion. So if we were to replace $\frac{27}{4}$ with rational $1<\beta<2$ (which ensures that the greedy base-$\beta$ expansion uses only coefficients in $\{0,1\}$), the bold statement above would be false.

There is of course a substantial difference between $1<\beta<2$ (when every real number has at least one base-$\beta$ expansion using only coefficients in $\{0,1\}$) and $\beta>2$ (when the set of real numbers that have a base-$\beta$ expansion using only coefficients in $\{0,1\}$ has measure zero). The greedy base-$\frac{27}{4}$ expansion of a rational number may use coefficients as large as $6$, so it's still possible that the statement in bold is true without contradicting Schmidt's theorem. But proving irrationality for even a single aperiodic expansion (let alone all of them) seems very difficult: for comparison, Terry Tao's blog post points out that the rationality of $\sum_{p\text{ prime}}\frac{1}{2^p-1}$ is unknown, and I would guess that $\sum_{p\text{ prime}}\frac{1}{(27/4)^p}$ is not any easier.

$\endgroup$

You must log in to answer this question.

Start asking to get answers

Find the answer to your question by asking.

Ask question

Explore related questions

See similar questions with these tags.